Download as pdf or txt
Download as pdf or txt
You are on page 1of 12

MAT102 Tutorial Solutions

Exercises 2.2
Determine the infinite limit:
x+2
25. limx→−3+ x+3

Solution: We have x → −3+ which means that x > −3. It follows x + 3 > 0. As the numerator of
the quotient tends to −1, and the denominator tends to 0 through positive numbers, the quotient
is always negative, if x is close to −3. Hence our guess for the limit is −∞. To prove this let K
be arbitrary, and we should choose δ > 0 such that if −3 < x < −3 + δ, then
x+2
< K.
x+3
We may assume that K < 0 and then the required inequality takes the form
x+2 x+3−1 1
K> = =1− ,
x+3 x+3 x+3
or
1 1 1
1−K < , or x + 3 < , or x < −3 + .
x+3 1−K 1−K
1 1
Let δ = 1−K
> 0 and we take any x with −3 < x < −3 + 1−K
. Then

1 1
0<x+3< , hence 1 − K < ,
1−K x+3
that is
x+2 1
=1− < K,
x+3 x+3
which was to be proved.
x−1
28. limx→0 x2 (x+1)

Solution: As x → 0, hence x − 1 → −1, the numerator is negative, and x2 → 0, x + 1 → 1, hence


the denominator is positive. Consequently, the quotient is always negative if x is close to 0. Hence
our guess for the limit is −∞. To prove this we apply estimation and the Squeeze Theorem: we
estimate the expression from above as follows:
1
x−1 2
−1 1 1
2
< 2
=− 2 < − 2.
x (x + 1) x (x + 1) 2x (x + 1) 4x

The explanation of this estimate is as follows: as x → 0, hence we may suppose that x < 12 which
proves the first inequality. For the second we observe that we have negative numbers on both
1
sides, hence to increase − 2x2 (x+1) we replace (x + 1) by 2. Finally, the right hand side − 4x12 tends
to −∞, and so does the smaller x2x−1 (x+1)
, by the Squeeze Theorem.
Exercises 4.4
Determine the limit at infinity:
1−x−x2
11. limx→−∞ 2x2 −7

Solution: As the degrees of the numerator and denominator are equal, we expect a finite limit.
Indeed, we simplify the expression by dividing both the top and the bottom by the highest degree
term: x2 to get
1
1 − x − x2 x2
− x1 − 1 1
lim 2
= lim 7 =− .
x→−∞ 2x − 7 x→−∞ 2 − x2 2
The explanation of this conclusion is as follows: in the numerator of the quotient (marked with
red circle) the terms x12 and x1 tend to zero as x → −∞, and the same holds for the term x72 in the
denominator. Consequently, by the Limit Laws, we conclude that the quotient tends to
0−0−1 1
=− .
2−0 2
We can get this result immediately by referring the rule: if in a rational function the degrees of
the polynomials in the top and in the bottom are equal, then the limit at ±∞ is equal to the
ratio of the leading coefficients.
x3 +5x
13. limx→∞ 2x3 −x2 +4

Solution: By the previous rule, the limit is 12 .



9x6 −x
18. limx→−∞ x3 +1

Solution: We want to use common square root for the top and the bottom. However, as the
denominator is negative for large negative values of x, hence we can write
p √
x3 + 1 = − (x3 + 1)2 = − x6 + 2x3 + 1,
and √ s r
9x6 − x 9x6 − x 9x6 − x
= − = − .
x3 + 1 (x3 + 1)2 x6 + 2x3 + 1
9
Under the square root, by the previous rule about leading coefficients, the limit is 1
= 9, and by
the Limit Laws, the limit is equal to −3.

19. limx→∞ 9x2 + x − 3x

2
Solution: We multiply the expression by 1 = √9x +x+3x
9x2 +x+3x
and apply the identity (a − b)(a + b) =
2 2
a − b to get

√ √ 9x2 + x + 3x
lim 9x2 + x − 3x = lim ( 9x2 + x − 3x) · √ =
x→∞ x→∞ 9x2 + x + 3x
√ √
( 9x2 + x − 3x)( 9x2 + x + 3x) (9x2 + x) − 9x2
lim √ = lim √ =
x→∞ 9x2 + x + 3x x→∞ 9x2 + x + 3x
x 1 1
lim √ = lim q = .
x→∞ 9x2 + x + 3x x→∞ 9 + 1 + 3 6
x

In the last step we simplify the quotient (marked by red circle) by dividing both the top and
bottom by x which appears under the square root as division by x2 , and finally we apply the Limit
Laws.
Exercises 2.3
Evaluate the limit if it exists:
t2 −9
15. limt→−3 2t2 +7t+3

Solution: Both the top and the bottom is zero at −3, hence both expressions can be factorized
by t − (−3) = t + 3. We have
t2 − 9 (t − 3)(t + 3) t−3 6
lim = lim = lim = .
t→−3 2t2 + 7t + 3 t→−3 (2t + 1)(t + 3) t→−3 2t + 1 5
In the last step we apply the Limit Law for quotients: if the limit of the denominator is different
from zero, then the limit is obtained by substitution.
x+2
19. limx→−2 x3 +8

Solution: Both the top and the bottom is zero at −2, hence both expressions can be factorized
by x − (−2) = x + 2. We have
x+2 x+2 1 1
lim 3 = lim 2
= lim 2 = .
x→−2 x + 8 x→−2 (x + 2)(x − 2x + 4) x→−2 x − 2x + 4 12
In the factorization we applied the identity
a3 − b3 = (a − b)(a2 + ab + b2 )
with the choice a = x, b = −2.
(2+h)3 −8
20. limh→0 h

Solution: Both the top and the bottom is zero at 0, hence both expressions can be factorized by
h − 0 = h. We have
(2 + h)3 − 8 8 + 12h + 6h2 + h3 − 8 12h + 6h2 + h3
lim = lim = lim = lim 12 + 6h + h2 = 12.
h→0 h h→0 h h→0 h h→0

In the first step we applied the identity


(a + b)3 = a3 + 3a2 b + 3ab2 + b3
with the choice a = 2, b = h.

Exercises 2.4
39. Find the limit, if it exists:
lim (2x + |x − 3|)
x→3

Solution: We check the left and right limits at 3: for the left limit let x < 3, then we have
x − 3 < 0, hence |x − 3| = −(x − 3) = 3 − x, and
lim (2x + |x − 3|) = lim− (2x + 3 − x) = lim− x + 3 = 6.
x→3− x→3 x→3

For the right limit let x > 3, then we have x − 3 > 0, hence |x − 3| = x − 3, and
lim (2x + |x − 3|) = lim− (2x + x − 3) = lim− 3x − 3 = 6.
x→3+ x→3 x→3

As both the left and right limit is 6, we have that the limit exists and
lim (2x + |x − 3|) = 6.
x→3
43. Find the limit, if it exists:
1 1 
lim− −
x→0 x |x|

Solution: As x → 0− hence x < 0, and |x| = −x. We have then


1 1  1 1 2
lim− − = lim− + = lim− = −∞.
x→0 x |x| x→0 x x x→0 x

44. Find the limit, if it exists:


1 1 
lim+ −
x→0 x |x|

Solution: As x → 0+ hence x > 0, and |x| = x. We have then


1 1  1 1
lim+ − = lim+ − = 0.
x→0 x |x| x→0 x x

49. If the symbol JxK denotes the greatest integer function, evaluate

(i) lim + JxK, (ii) lim JxK, (iii) lim JxK


x→−2 x→−2 x→−2.4

Solution:
(i) If x → −2+ , then x > −2, and we may assume that x < −1, hence JxK = −2. It follows

lim JxK = −2.


x→−2+

(ii) If x → −2− , then x < −2, and we may assume that x > −3, hence JxK = −3. It follows

lim JxK = −3.


x→−2−

As limx→−2− JxK 6= limx→−2+ JxK, the limit does not exist.


(iii) If x → −2.4, then we may assume that −3 < x < −2, hence JxK = −3. It follows

lim JxK = −3.


x→−2.4

51. If f (x) = JxK + J−xK, show that limx→2 f (x) exists but it is not equal to f (2).

Solution: As x → 2, we may assume that 1.5 < x < 2.5. We check the one-sided limits at 2. For
the left limit let x < 2, then −2 < −x < −1.5, hence

JxK = 1, J−xK = −2, and f (x) = JxK + J−xK = −1.

For the right limit let x > 2, then −2.5 < x < −2, hence

JxK = 2, J−xK = −3, and f (x) = JxK + J−xK = −1.

It follows that the limit exists, and it is −1. On the other hand, f (2) = 2 + (−2) = 0.
60. Evaluate √
6−x−2
lim √
x→2 3−x−1

Solution:
√ We√ apply the method of rationalization, multiplying both the top and bottom by
( 6 − x + 2)( 3 − x + 1) to get
√ √ √ √
6−x−2 ( 6 − x − 2)( 6 − x + 2)( 3 − x + 1)
lim √ = lim √ √ √ =
x→2 3 − x − 1 x→2 ( 3 − x − 1)( 6 − x + 2)( 3 − x + 1)
√ √ √
((6 − x) − 4)( 3 − x + 1) (2 − x)( 3 − x + 1) 3−x+1 1
lim √ = lim √ = lim √ = .
x→2 ((3 − x) − 1)( 6 − x + 2) x→2 (2 − x)( 6 − x + 2) x→2 6−x+2 2
In the last step we substitute x = 2, as th function is continuous at this point.

61. Is there a number a such that


3x2 + ax + a + 3
lim
x→−2 x2 + x − 2
exists? If so, find the value of a and the value of the limit.

Solution: For x 6= −2 can write the expression in the form


3x2 + ax + a + 3 a−3 15 − a
lim 2
= lim 3 + + .
x→−2 x +x−2 x→−2 x − 1 (x − 1)(x + 2)
The first two terms are continuous at −2, hence we can write, by substituting x = −2
3x2 + ax + a + 3 a−3 15 − a
lim 2
=3− + lim .
x→−2 x +x−2 3 x→−2 (x − 1)(x + 2)
If a = 15, then the last expression is zero, hence we have, by substituting a = 15
3x2 + ax + a + 3 3x2 + 15x + 18 3(x + 3)(x + 2) 3(x + 3)
lim 2
= lim = lim = lim = −1.
x→−2 x +x−2 x→−2 (x − 1)(x + 2) x→−2 (x − 1)(x + 2) x→−2 x − 1

If a < 15, then 15 − a > 0, and x − 1 → −3, x + 2 → 0 as x → −1, hence


15 − a
lim − = +∞,
x→−2 (x − 1)(x + 2)
and
15 − a
lim + = −∞,
x→−2 (x − 1)(x + 2)
hence the limit does not exist. If a > 15, then 15 − a < 0, hence similarly
15 − a
lim − = −∞,
x→−2 (x − 1)(x + 2)
and
15 − a
lim + = +∞,
x→−2 (x − 1)(x + 2)
and the limit does not exist. Hence the only number a for which the limit exists is a = 15, and in
that case the limit is −1.
Exercises 2.5

11. Use the definition of continuity and the properties of limits to show that the function is
continuous at the given number a.


f (x) = x2 + 7 − x, a = 4


Solution: The function x 7→ x2 and the function x 7→ 7 − x is continuous at a = 4, hence their
limits√can be computed by substitution; then we apply the Limit Law for the sum: limx→a f (x) =
16 + 3 = f (a). It follows that f is continuous at a.

f (x) = (x + 2x3 )4 , a = −1

Solution: Argument is same as above, f can be obtained from the continuous function x 7→ x by
repeated application of the Limit Laws.

2t − 3t2
h(t) = , a=1
1 + t3

Solution: Argument is same as above, h can be obtained from the continuous function t 7→ t by
repeated application of the Limit Laws.

39. Find the numbers at which f is discontinuous. At which of these numbers is f continuous
from the right, from the left, or neither? Sketch the graph of f .

x + 2 if x < 0

f (x) = 2x2 if 0 ≤ x ≤ 1

2 − x if x > 1

Solution: We start with the graph of f :


2.0

1.5

1.0

0.5

-2 -1 1 2

The function is clearly continuous at any point a different from 0 and 1, as it is equal to a continuous
function in an open interval around a. At the point a = 0 we have

lim f (x) = lim− x + 2 = 2,


x→0− x→0
and
lim f (x) = lim− 2x2 = 0,
x→0+ x→0

further f (0) = 0. It follows that f is continuous from the right at 0, and it is discontinuous from
the left at 0. Hence the point a = 0 is a non-removable discontinuity.

At the point a = 1 we have


lim f (x) = lim− 2x2 = 2,
x→1− x→1

and
lim f (x) = lim+ 2 − x = 1,
x→1+ x→1

further f (1) = 2. It follows that f is continuous from the left at 1, and it is discontinuous from
the right at 1. Hence the point a = 1 is a non-removable discontinuity.

47. Use the Intermediate Value Theorem to show that there is a root of the given equation in the
specified interval.
x4 + x − 3 = 0, (1, 2)

Solution: Let f (x) = x4 + x − 3, then f is a continuous function on the whole real line. We have
f (1) = −1 < 0, and f (2) = 15 > 0. By the Intermediate Value Theorem, f takes every value
between f (1) and f (2) in the interval (1, 2), hence there is a root c with 1 < c < 2 such that
c4 + c − 3 = 0.

61. Is there a number that is exactly 1 more than its cube?

Solution: Such a number must satisfy x = x3 + 1, or x3 − x + 1 = 0. Let f (x) = x3 − x + 1, then


f is a continuous function on the whole real line. Moreover, f (−2) = −5 < 0, and f (1) = 1 > 0.
By the Intermediate Value Theorem, f takes every value between f (−2) and f (1) in the interval
(−2, 1), hence there is a number c between −2 and 1 that is exactly 1 more than its cube.

63. Show that the function (


x4 sin(1/x) if x 6= 0
f (x) =
0 if x = 0
is continuous on (−∞, +∞).

Solution: The function f is continuous at every point a different from 0, as it is equal to a


continuous function in some open interval around a, by the Limit Laws.

We show that f is continuous at a = 0. For this it is enough to show that limx→0 f (x) = 0. We
apply the Squeeze Theorem: clearly, for each x 6= 0 we have

|f (x)| = x4 sin(1/x) = |x|4 · sin(1/x) ≤ |x|4 ,

that is
−|x|4 ≤ f (x) ≤ |x|4 ,

which implies our statement, as limx→0 |x|4 = 0. We used the fact that sin(1/x) ≤ 1.
Review – True-False quiz
Determine whether the statement is true or false. If it is true, explain why. If it is false, explain
why or give an example that disproves the statement.

1.  2x 8  2x 8
lim − = lim − lim
x→4 x − 4 x−4 x→4 x − 4 x→4 x − 4

Solution: The statement is FALSE: the two limits on the right side do not exist.

2.
x2 + 6x − 7 limx→1 x2 + 6x − 7
lim =
x→1 x2 + 5x − 6 limx→1 x2 + 5x − 6

Solution: The statement is FALSE: the right hand side makes no sense.

3.
x−3 limx→1 x − 3
lim =
x→1 x2 + 2x − 4 limx→1 x2 + 2x − 4

Solution: The statement is TRUE: it follows from the Limit Law for quotients.

4. If limx→5 f (x) = 2 and limx→5 g(x) = 0, then limx→5 [f (x)/g(x)] does not exist.

Solution: The statement is FALSE: if f (x) = 2 and g(x) = (x − 5)2 , then

f (x) 2
lim = lim = +∞.
x→5 g(x) x→5 (x − 5)2

5. If limx→5 f (x) = 0 and limx→5 g(x) = 0, then limx→5 [f (x)/g(x)] does not exist.

Solution: The statement is FALSE: if f (x) = x − 5 and g(x) = x − 5, then

f (x) x−5
lim = lim = 1.
x→5 g(x) x→5 x − 5

6. If limx→6 [f (x)g(x)] exists, then the limit must be f (6)g(6).


1
Solution: The statement is FALSE: if f (x) = x − 6 and g(x) = x−6
, then

f (x) x−6
lim = lim = 1,
x→6 g(x) x→6 x − 6

however g(6) is not defined.

7. If p is a polynomial, then limx→b p(x) = p(b).

Solution: The statement is TRUE: every polynomial is continuous, hence the limit at every
point is obtained by substitution.
8. If limx→0 f (x) = +∞ and limx→0 g(x) = +∞, then limx→0 [f (x) − g(x)] = 0.
1 1
Solution: The statement is FALSE: if f (x) = 1 + x2
and g(x) = x2
, then

lim [f (x) − g(x)] = lim 1 = 1.


x→0 x→0

9. If the line x = 1 is a vertical asymptote of y = f (x), then f is not defined at 1.


1
Solution: The statement is FALSE: if f (x) = (x−1) 2 for x 6= 1 and f (1) = 0, then

limx→1 f (x) = +∞, hence the line x = 1 is a vertical asymptote of y = f (x), however, f
is defined at 1.

10. If f (1) > 0 and f (3) < 0, then there exists a number c between 1 and 3 such that f (c) = 0.

Solution: The statement is FALSE: if f (x) = 1 for x ≤ 2 and f (x) = −1 for x > 2, then
there is no such c.

11. If f is continuous at 5 and f (5) = 2 and f (4) = 3, then limx→2 f (4x2 − 11) = 2.

Solution: The statement is TRUE: the function

g(x) = f (4x2 − 11)

is a composition of the continuous function x 7→ 4x2 − 11 and the function f , which is


continuous at 5. On the other hand, if x → 2, then 4x2 − 11 → 5, hence the composite
function is continuous at 2, which implies that

lim f (4x2 − 11) = lim g(x) = g(2) = f (5) = 2.


x→2 x→2

12. If f is continuous on [−1, 1] and f (−1) = 4 and f (1) = 3, then there exists a number r such
that |r| < 1 and f (r) = π.

Solution: The statement is TRUE: by the Intermediate Theorem, on the interval (−1, 1)
the continuous function f takes every value between 3 and 4. As 3 < π < 4, there is an r
with −1 < r < 1 such that f (r) = π. Observe, that −1 < r < 1 means |r| < 1.

13. Let f be a function such that limx→0 f (x) = 6. Then there exists a number δ such that if
0 < |x| < δ, then |f (x) − 6| < 1.

Solution: The statement is TRUE: choose ε = 1 in the definition of the limit.

14. If f (x) > 1 for all x and limx→0 f (x) exists, then limx→0 f (x) > 1.

Solution: The statement is FALSE: let f (x) = 1 + x2 for x 6= 0, and f (0) = 2. Then
f (x) > 1 for all x, and limx→0 f (x) = 1.

15. The equation x10 − 10x2 + 5 = 0 has a root in the interval (0, 2).

Solution: The statement is TRUE: if f (x) = x10 − 10x2 + 5, then f (0) = 5 > 0, and
f (1) = −4 < 0 and f is continuous, hence f (c) = 0 for some c between 0 and 1.
Exercises 3.2
Find the derivative of the function using the definition of derivative. State the domain of the
function and the domain of its derivative:

23. g(x) = 1 + 2x

Solution: The domain of the function is the set of all x’s with 1 + 2x ≥ 0, that is x ≥ − 12 . For
the derivative we compute the limit
√ √
g(x) − g(a) 1 + 2x − 1 + 2a
lim = lim .
x→a x−a x→a x−a
Here a ≥ − 21 , and for a = − 21 the above limit is meant as right limit. We rationalize the numerator:
√ √ √ √ √ √
1 + 2x − 1 + 2a 1 + 2x − 1 + 2a 1 + 2x + 1 + 2a
lim = lim ·√ √ =
x→a x−a x→a x−a 1 + 2x + 1 + 2a

(1 + 2x) − (1 + 2a) 2(x − a)


lim √ √ = lim √ √ =
x→a (x − a)( 1 + 2x + 1 + 2a) x→a (x − a)( 1 + 2x + 1 + 2a)
2 1
lim √ √ = .
x→a 1 + 2x + 1 + 2a 1 + 2a
Observe that this is not defined at a = − 21 . Hence the derivative of g is

1 1
g 0 (x) = , defined for x > − .
1 + 2x 2

4t
25. G(t) = t+1

Solution: The domain of the function is the set of all t’s with t + 1 6= 0, that is t 6= −1. For the
derivative we compute the following limit for a 6= 1:
4t 4a
G(t) − G(a) t+1
− a+1 4(t − a) 4 4
lim = lim = lim = lim = .
t→a t−a x→a t−a t→a (t + 1)(a + 1)(t − a) t→a (t + 1)(a + 1) (a + 1)2

Hence the derivative of G is


4
G0 (t) = , defined for t 6= −1.
(t + 1)2

49. Show that the function f (x) = |x − 6| is not differentiable at 6. Find a formula for f 0 and
sketch its graph.

Solution: We compute the left and right limits of

f (x) − f (6)
x−6
at the point 6. For the left limit we have x < 6, and |x − 6| = −(x − 6), hence

f (x) − f (6) −(x − 6)


lim− = lim− = −1,
x→6 x−6 x→6 x−6
and similarly, for the right limit we have x > 6, and |x − 6| = x − 6, hence

f (x) − f (6) x−6


lim+ = lim+ = 1.
x→6 x−6 x→6 x − 6

It follows that the limit


f (x) − f (6)
lim
x→6 x−6
does not exist, hence the function f is not differentiable at 6. We have
(
−1 for x < 6
f 0 (x) =
1 for x > 6.

The graph of f 0 is as follows:


1.0

0.5

5.5 6.0 6.5 7.0

-0.5

-1.0

51.

a) Sketch the graph of the function f (x) = x|x|.


b) For what values of x is f differentiable?
c) Find a formula for f 0 .

Solution: a) The graph of f is as follows:


1.0

0.5

-1.0 -0.5 0.5 1.0

-0.5

-1.0

b) We have to calculate the limit


x|x| − a|a|
lim .
x→a x−a
Suppose that a > 0, then if x is close to a, then x > 0 and |a| = a, |x| = x, hence

x|x| − a|a| x2 − a2 |a|


lim = lim = lim (x + a) = 2a = 2|a|.
x→a x−a x→a x−a x→a
If a < 0, then, by the same argument x < 0 and we have |a| = −a, |x| = −x, hence
x|x| − a|a| −x2 + a2 |a|
lim = lim = lim −(x + a) = −2a = 2|a|.
x→a x−a x→a x−a x→a

Finally, if a = 0, then
x|x| − a|a| x|x|
lim = lim = lim |x| = 0 = 2|a|.
x→a x−a x→0 x x→0

Consequently, for all values of a we have that f 0 (a) exists and f 0 (a) = 2|a|.

c) In other words, f 0 (x) = 2|x|.


Exercises 3.3
Differentiate.
 
25. F (y) = y12 − 3
y4
(y + 5y 3 )

Solution: The easiest way is to expand the product and then differentiate the sum term by term,
applying the differentiation rule for general power functions:
1 3 5
F (y) = + 5y − 4 − = y −1 + 5y − 3y −4 − 5y −1 ,
y y y
hence
4 12
F 0 (y) = −y −2 + 5 + 12y −5 + 5y −2 = 5 + 2 + 5 .
y y
x
41. f (x) = x+ xc

Solution: Here we have a quotient, and also in the denominator there is another quotient:
0 (x)0 (x + xc ) − x(x + xc )0 x + xc − x(1 − xc2 ) x
c
+ xc 2c
f (x) = c 2 = c 2 = c 2 = .
(x + x ) (x + x ) (x + x ) x(x + xc )2
53. Find an equation of the tangent line and normal line to the curve at the given point:

y = x + x, (1, 2).

Solution: The equation of the tangent line at the point (a, f (a)) is
y − f (a) = f 0 (a)(x − a),
and the equation of the normal line at the point (a, f (a)) is
1
y − f (a) = − 0 (x − a),
f (a)
assuming that f 0 (a) exists and is different from 0. We have

f (x) = y = x + x = x + x1/2 ,
hence
1 1
f 0 (x) = 1 + x−1/2 = 1 + √ ,
2 2 x
and a = 1, f (a) = 2, consequently the two equations are:
1 3 1
y − 2 = 1 + (x − 1), or y = x + (tangent line),
2 2 2
and
1 2 8
y − 2 = − 1 + )−1 (x − 1), or y = − x + (normal line).
2 3 3

You might also like